Đến nội dung

nhungvienkimcuong

nhungvienkimcuong

Đăng ký: 23-12-2014
Offline Đăng nhập: Riêng tư
****-

#742017 Chứng minh rằng $\frac{\left ( 2k \right )!...

Gửi bởi nhungvienkimcuong trong 05-11-2023 - 12:06

Với các số nguyên $a,b$ nguyên tố cùng nhau, $a> b> 1$, ta xét dãy số sau: 

$u_n=\varphi (a^{2n-1}+b^{2n-1})$ với $n=1,2,3,...$ 

1. Chứng minh rằng nếu $p> 3$ là số nguyên tố lẻ và có số hạng nào đó của dãy trên bằng $2p$ thì $a+b=2p+1$ hoặc $a+b=2(2p+1)$

2. Chứng minh rằng $\frac{\left ( 2k \right )!}{k!}\mid \prod_{i=1}^{k}u_i$

Sử dụng kết quả này là giải quyết được.

Ngoài ra ở ý đầu tiên thì chú ý rằng nếu $\varphi(x)=2p$ thì $x\in \{q^k,2q^k\}$ với $q$ là số nguyên tố và $k$ là số nguyên dương, từ đây lập luận được rằng $k=1$ và $q=2p+1$.




#741988 gtnn $x+y+z$

Gửi bởi nhungvienkimcuong trong 03-11-2023 - 04:05

Cho $x,y,z$ thực thỏa mãn $3\leq x,y,z\leq 5$ và $x^2+y^2+z^2=50$

Tìm gtnn $x+y+z$

Cách 1.

Khai triển $(x-3)(y-3)(z-3)\ge 0$ ta có

\begin{equation*} xyz-3(xy+yz+zx)+9(x+y+z)-27\ge 0.\tag{1}\end{equation*}

Khai triển $(x-5)(y-5)(z-5)\le 0$ ta có

\begin{equation*}xyz-5(xy+yz+zx)+25(x+y+z)-125\le 0.\tag{2}\end{equation*}

Lấy $(1)$ trừ $(2)$ ta có được

\begin{equation*}2(xy+yz+zx)-16(x+y+z)+98\ge 0.\tag{3}\end{equation*}

Mặt khác $2(xy+yz+zx)=(x+y+z)^2-(x^2+y^2+z^2)=(x+y+z)^2-50$ nên $(3)$ tương đương

\[(x+y+z)^2-16(x+y+z)+48\ge 0\implies x+y+z\ge 12.\]

Dấu bằng xảy ra khi và chỉ khi $x=3,y=4,z=5$ cùng các hoán vị.

 

Cách 2.

Không mất tính tổng quát ta sắp xếp $x\ge y\ge z$. Giả sử

\[x+y+z<12\implies x+y<12-z\le 9.\]

Sử dụng khai triển Abel ta có

\begin{align*}x^2+y^2+z^2=x\cdot x+y\cdot y+z\cdot z&=x(x-y)+(x+y)(y-z)+(x+y+z)z\\&<5(x-y)+9(y-z)+12z\\&=5x+4y+3z\\&=x+4(x+y+z)-z\\&< 5+4\cdot 12-3 =50.\end{align*}
Từ mâu thuẫn này ta có $x+y+z\ge 12$.



#741951 $\lim_{n\to \infty }\sqrt[n]{1+cos(2n...

Gửi bởi nhungvienkimcuong trong 31-10-2023 - 21:09

Tính giới hạn sau:

$\lim_{n\to \infty }\sqrt[n]{1+cos(2n)}$

Nhìn lướt qua thì cảm giác đây là bài toán bình thường cho phổ thông, đến lúc đặt bút thì chịu  :icon6: . Tìm trên mạng thấy xuất hiện ở đây.




#741916 $x_0=\frac{1}{2} , x_{n+1}=x_n+\...

Gửi bởi nhungvienkimcuong trong 29-10-2023 - 20:49

Bài toán:  Cho dãy số thực $(x_n)_n$ được định nghĩa như sau

 

$$x_0=\frac{1}{2} \, \, , \, \, x_{n+1}=x_n+\frac{x^{2}_{n}}{2017} \,\, , \,\, \forall n\in \mathbb{N}$$

 

Tìm số tự nhiên $k$ nhỏ nhất sao cho $x_k>1$

 

Note:  Đây là bài toán cũ không có lời giải của diễn đàn, thấy thú vị nên post lại cho các bạn giải thử. Mình thì tìm được $k=4034$, bạn nào tìm được giá trị nhỏ hơn thì cho lời giải nhé.

Dựa vào giả thiết thấy rằng hướng đi là cần tìm một sai phần phù hợp. Ta có

$$\frac{1}{x_{n+1}}=\frac{2017}{x_n(x_n+2017)} = \frac{1}{x_n}-\frac{1}{x_n+2017}.$$

Từ đây dẫn đến

$$2-\frac{1}{x_n}=\sum_{i=0}^{n-1}\frac{1}{x_i+2017}.$$

$\bullet$ Chứng minh $x_k<1$ với mọi $k\le 2017$.

Dễ thấy $x_i>0$ với mọi $i<k$, do vậy

\[2-\frac{1}{x_k}=\sum_{i=0}^{k-1}\frac{1}{x_i+2017}<\sum_{i=0}^{k-1}\frac{1}{2017}=\frac{k}{2017}\le 1.\]

$\bullet$ Chứng minh $x_{2018}>1$.

Vì $x_i<1$ với mọi $i<2018$ nên

\[2-\frac{1}{x_{2018}}=\sum_{i=0}^{2017}\frac{1}{x_i+2017}>\sum_{i=0}^{2017}\frac{1}{2018}= 1.\]

Vậy $k=2018$ là giá trị cần tìm (Đáp số được kiểm định bởi python).

n = 0
xn = 1/2

while xn < 1:
    xn = xn + xn ** 2 / 2017
    n += 1

print(n)



#741910 $e^{A+B}=e^A\,e^B=e^B\,e^A$

Gửi bởi nhungvienkimcuong trong 29-10-2023 - 17:04

Mình từng làm bài này khi còn là sinh viên đại học. File gửi kèm  giaitichham.pdf   107.72K   54 Số lần tải

 




#741870 $x^{2021}+y!=y^{2021}+x!$

Gửi bởi nhungvienkimcuong trong 27-10-2023 - 20:32

 Chứng minh rằng không tồn tại các số nguyên dương $x,y>2$ phân biệt sao cho:

$$x^{2021}+y!=y^{2021}+x!$$

Bài này ý tưởng rõ ràng, tuy nhiên lúc làm chi tiết cũng khá mắc công.

 

Đặt $m=2021$ và giả sử $x<y$. Lời giải xuất phát từ nhận xét: xét $p$ là ước nguyên tố bất kì của $x$, từ giả thiết suy ra $p\mid y$, do vậy

\begin{equation}\label{a} m\le v_p(y^m-x^m)=v_p(y!-x!)=v_p(x!)<\frac{x}{p-1}.\end{equation} 

Mệnh đề

Xét hàm số $f\colon \mathbb{N}^*\to \mathbb{Z}$ được xác định bởi $f(k)=k^m-k!$. Khi đó hàm $f$ nhận giá trị âm và giảm ngặt trong khoảng $(2m-1,+\infty)$.

Xét trường hợp $p\ge 3$, từ \eqref{a} ta có $x>2m$, kết hợp với Theorem suy ra $f(x)> f(y)$.

Vậy chỉ còn trường hợp $x$ là lũy thừa của $2$, cũng từ \eqref{a} thì $x>m$. Đương nhiên $y$ chẵn, xét $q$ là một ước nguyên tố lẻ của $y$.

  • Nếu $q\le x$, từ giả thiết suy ra $q\mid x$ (vô lí).
  • Với $q>x$ suy ra $y\ge 2q>2x>2m$, theo Theorem thì $f(y)<0$. Từ đây dễ thấy $f(x)$ âm hay dương cũng vô lí (trường hợp âm kết hợp với phần chứng minh của Theorem).

 

 

Ghi chú. Một số bài toán tương tự có thể kể đến như sau

Bài 1 (Trung Âu 2015). Tìm tất cả cặp số nguyên dương $(a,b)$ thỏa mãn

$$a!+b!=a^b+b^a.$$

Bài 2. Tìm tất cả các số nguyên dương $n$ sao cho $(n-1)!$ là bội của $n^2$.

Bài 3 (IMO 2019). Tìm tất cả các cặp số nguyên dương $(k,n)$ sao cho

$$k!=(2^n-1)(2^n-2)(2^n-4)\cdots (2^n-2^{n-1}).$$




#741846 $\frac{m_a}{h_{a}}+\frac{m_...

Gửi bởi nhungvienkimcuong trong 26-10-2023 - 20:37

 

Trong  tam giác $ABC$ nhọn có $h_{a}, h_{b},h_{c}$ là độ dài các đường cao xuất phát từ các đỉnh $A,B,C$.  $m_{a},m_{b},m_{c}$, là độ dài các đường trung tuyến hạ từ các đỉnh $A,B,C$.  $(O;R),  (O';r)$ là tâm đường tròn ngoại tiếp và nội tiếp tam giác $ABC$. CMR : $\frac{m_{a}}{h_{a}}+\frac{m_{b}}{h_{b}}+\frac{m_{c}}{h_{c}}\leq \frac{R+r}{r}$

Gọi $A',B',C'$ lần lượt là trung điểm các đoạn $BC,CA,AB$. Kí hiệu $S$ là diện tích của tam giác $ABC$, chú ý rằng

\[m_a=AA'\le OA+OA'=R+OA',\quad \frac{(a+b+c)r}{2}=\frac{a\cdot OA'+b\cdot OB'+c\cdot OC'}{2}=S.\]

Do vậy

\[\sum\frac{m_a}{h_a}=\sum\frac{am_a}{ah_a}\le \sum\frac{a(R+OA')}{2S}=R\frac{\sum a}{2S}+\frac{\sum a\cdot OA'}{2S}=\frac{R}{r}+1.\]

Screenshot 2023-10-26 223602.png




#741783 Tìm a,b nguyên dương thỏa mãn $a^{4}+10a^{2}+2^...

Gửi bởi nhungvienkimcuong trong 18-10-2023 - 18:26

Tìm $a,b \in Z , a,b>0$ thỏa mãn $a^{4}+10a^{2}+2^{b}$ là 1 số chính phương

Vì bài này để ở THCS nên mình sẽ nêu lại một số ý cần thiết liên quan tới số mũ đúng. Tài liệu tham khảo: Phạm Quang ToànAmir Hosein.

Mệnh đề

Với mỗi số nguyên dương $x$, kí hiệu $v_2(x)$ là số tự nhiên $k$ lớn nhất sao cho $2^k\mid x$ (ví dụ $v_2(12)=2$ và $v_2(15)=0$). Với $x,y$ là các số nguyên dương thì ta có

  1. $v_2(x^y)=y\cdot v_2(x)$, do vậy $v_2(x^2)$ luôn là số chẵn.
  2. $v_2(x+y)= \min\big(v_2(x),v_2(y)\big)$ nếu $v_2(x)\neq v_2(y)$.

 

Quay lại bài toán. Xét $b\ge 5$, nếu $a$ lẻ thì 

\[a^2\equiv 1\pmod{4}\implies a^4+10a^2+2^b\equiv 3\pmod{4}.\]

Điều này vô lí thì số chính phương khi chia cho $4$ chỉ dư $0$ hoặc $1$, do vậy $a$ chẵn. Nếu $b\ge 2v_2(a)+2$ thì theo Mệnh đề 2 ta có

\[v_2(a^4+10a^2+2^b)=v_2(10a^2)=2v_2(a)+1,\]

vô lí theo Mệnh đề 1. Vậy $b\le 2v_2(a)+1$, suy ra

\[a^4+10a^2+2^b\le a^4+12a^2<(a^2+6)^2.\]

Mặt khác $b\ge 5$ nên $a^4+10a^2+2^b>(a^2+5)^2$. Vậy $b\ge 5$ thì không tồn tại $a$, còn lại chỉ cần xét $b\in\{1,2,3,4\}$ thì đơn giản rồi.




#741779 $ \sqrt{a^2+16bc} + \sqrt{b^2+16ca} +...

Gửi bởi nhungvienkimcuong trong 18-10-2023 - 11:17

Cho $a,b,c \ge 0 thoả ab+bc+ca>0$.Chứng minh rằng:
$ \sqrt{a^2+16bc} + \sqrt{b^2+16ca} + \sqrt{c^2+16ab} \ge 6\sqrt{ab+bc+ca} $

Cơ bản thì lời giải của mình khá giống anh Quý, khác là mình may mắn tìm được số $24$ ở \eqref{ab} nên có phần ngắn hơn.

 

Về mặt ý tưởng, ta mong muốn làm giảm số căn thức lại bằng cách sử dụng một bất đẳng thức phụ. Giả sử $c=\min(a,b,c)$, mình tìm được kết quả sau

\begin{equation}\label{ab}\sqrt{a^2+16bc}+\sqrt{b^2+16ca}\ge \sqrt{(a+b)^2+{\color{Red} 24}(a+b)c}.\end{equation}

Áp dụng \eqref{ab} kết hợp với $\sqrt{c^2+16ab}\ge 4\sqrt{ab}$, do vậy cần chứng minh

\[\sqrt{(a+b)^2+24(a+b)c}\ge 6\sqrt{ab+bc+ca}-4\sqrt{ab}.\]

Bình phương hai vế tương đương với

\begin{equation}(a+b)^2+48\sqrt{ab(ab+bc+ca)}\ge 52ab+12c(a+b).\end{equation}

Ta thấy rằng $ab(ab+bc+ca)-\left(ab+\frac{c(a+b)}{3}\right)^2=\dfrac{c(a+b)(3ab-ac-bc)}{9} \geq 0$Do vậy

\[VT(2)\ge 4ab+48\left(ab+\frac{c(a+b)}{3} \right )=52ab+16c(a+b)\ge VP(2).\]

 

 

Ghi chú. Một số bài toán có vẻ tương tự ở đâyđây.




#741736 Công thức tính mode của mẫu dữ liệu ghép nhóm

Gửi bởi nhungvienkimcuong trong 15-10-2023 - 16:34

Cố gắng chứng minh công thức $\eqref{1}$ bằng hình học, ta thấy rằng, hợp lý nhất thì mode phải là hoành độ của giao điểm $D$ của $AC$ và $FH$. Đặt $x=CE$.

attachicon.gif screenshot_1697216271.png

Việc lấy giao điểm $AC\cap FH$ là chưa chính xác, vì điểm đạt được mode phải thỏa mãn "tỉ lệ". Như vậy để thu được kết quả đúng thì cần lấy giao điểm $BF\cap CG$.

 

Nếu coi mẫu dữ liệu là liên tục thì mode là điểm cực đại của hàm phân phối $F(x)$. Bài toán trở thành:
Cho hàm số $F(x)$ liên tục trên $[a_1;a_4]$. Chia đoạn $[a_1;a_4]$ thành ba đoạn bằng nhau $[a_1; a_2], [a_2; a_3]; [a_3; a_4]$. Đặt $$\sum_{x=a_i}^{a_{i+1}}F(x)=m_i, \quad i = 1,2,3.$$
Biết rằng $m_2>m_1; m_2>m_3$. Tìm điểm cực đại của $F(x)$.

Điểm cực đại sẽ là nghiệm của phương trình $F'(x)=0$.




#741698 Bài toán về giới hạn trong đề thi HSG cấp trường

Gửi bởi nhungvienkimcuong trong 12-10-2023 - 19:55

Gợi ý: chứng minh $u_n>n^2$ bằng phương pháp quy nạp, từ đó suy ra $u_n<(n+1)^2$. Như vậy $n^2<u_n<(n+1)^2$.




#741675 GTNN của $A= \frac{1}{a^{2}+b^{2...

Gửi bởi nhungvienkimcuong trong 09-10-2023 - 22:26

Tìm GTNN của A biết $a,b,c \ge 0: a+b+c \le 3$ và $A= \frac{1}{a^{2}+b^{2}}+\frac{1}{b^{2}+c^{2}}+\frac{1}{c^{2}+a^{2}}$

Dấu bằng xảy ra khi một biến bằng $0$ và hai biến còn lại bằng nhau, ta sẽ chứng minh

\[ \frac{1}{a^{2}+b^{2}}+\frac{1}{b^{2}+c^{2}}+\frac{1}{c^{2}+a^{2}}\ge\frac{10}{(a+b+c)^2}.\]

Ý tưởng bài này là giả sử $c=\min(a,b,c)$, khi đó

\[a^2+b^2\le \left(a+\frac{c}{2}\right)^2+\left(b+\frac{c}{2}\right)^2,\quad b^2+c^2\le \left(b+\frac{c}{2}\right)^2,\quad c^2+a^2\le \left(a+\frac{c}{2}\right)^2.\]

Phần còn lại chỉ cần chứng minh $\frac{1}{x^2+y^2}+\frac{1}{y^2}+\frac{1}{x^2}\ge \frac{10}{(x+y)^2}$ với $x=a+\frac{c}{2}$ và $y=b+\frac{c}{2}$. Bài toán hai biến này thì không quá khó, bạn tự giải quyết nhé.

 

 

Ghi chú. Một số bài toán hoàn toàn tương tự, tham khảo ở đâyđây.




#741651 $a_{n+1}=\sqrt{2-\sqrt{4-a_{n}^...

Gửi bởi nhungvienkimcuong trong 07-10-2023 - 18:14

Cho $a_{0}=\sqrt{2}$,$b_{0}=2$,$a_{n+1}=\sqrt{2-\sqrt{4-a_{n}^{2}}}$,$b_{n+1}=\frac{2b_{n}}{2+\sqrt{4+b_{n}^{2}}}$

1. Chứng minh $(a_{n},b_{n})$ không tăng và hội tụ về 0

2. Chứng minh $(2^{n}a_{n})$ tăng và $(2^{n}b_{n})$ giảm và hai dãy đó hội tụ về cùng một giá trị

3. Chứng minh rằng tồn tại một số C sao cho với mọi n thì bất đẳng thức sau luôn đúng:

$0<b_{n}-a_{n}<\frac{C}{8^{n}}$

Với hình thức truy hồi như này thì nghĩ ngay đến công thức lượng giác, mò một tí ta thấy rằng với $0<\alpha<\frac{\pi}{2}$ thì

\[\sqrt{2-\sqrt{4-(2\sin \alpha)^2}}=2\sin\frac{\alpha}{2},\quad \frac{2\cdot 2\tan\alpha}{2+\sqrt{4+(2\tan\alpha)^2}}=2\tan\frac{\alpha}{2}.\]

Từ đây quy nạp thu được $a_n=2\sin\frac{\pi}{2^{n+2}}$ và $b_n=2\tan\frac{\pi}{2^{n+2}}$.

 

Có được công thức của hai dãy thì dễ dàng xử lí hai ý đầu tiên, ý còn lại tương đương với $0<\frac{\tan\frac{\pi}{2^{n+2}}-\sin\frac{\pi}{2^{n+2}}}{\left ( \frac{\pi}{2^{n+2}} \right )^3}<C'$. Chú ý rằng

\[\lim_{n\to \infty}\frac{\tan\frac{\pi}{2^{n+2}}-\sin\frac{\pi}{2^{n+2}}}{\left ( \frac{\pi}{2^{n+2}} \right )^3}=\lim_{x\to 0}\frac{\tan x-\sin x}{x^3}=\frac{1}{2}\]

là giải quyết xong.




#741611 $ab^2c^3 \ge 1$

Gửi bởi nhungvienkimcuong trong 04-10-2023 - 19:01

Cho $a,b,c>0$ thoả $a+b+c=\frac{1}{a} + \frac{1}{b} + \frac{1}{c}$. Chứng minh rằng nếu $a \le b \le c$ thì $ab^2c^3 \ge 1$

Để bỏ đi điều kiện đẳng thức của giả thiết ta đưa về bất đẳng thức thuần nhất, nghĩa là chứng minh

\[ab^2c^3\ge \left(\frac{abc(a+b+c)}{ab+bc+ca} \right )^3\iff (ab+bc+ca)^3\ge a^2b(a+b+c)^3.\]

Theo bất đẳng thức Cô-si thì $a^2b\le \left(\frac{2a+b}{3}\right)^3$, do đó ta chỉ cần chứng minh $ab+bc+ca\ge \frac{2a+b}{3}\cdot(a+b+c)$. Bất đẳng thức cuối luôn đúng vì

\begin{align*}3(ab+bc+ca)-(2a+b)(a+b+c)&=c(2b+a)-b^2-2a^2\\ &\ge b(2b+a)-b^2-2a^2=(b-a)(b+2a)\ge 0.\end{align*}




#741564 $ S=\begin{Bmatrix} 1,2,3,...,2n \end{Bmatrix...

Gửi bởi nhungvienkimcuong trong 29-09-2023 - 21:56

Với mỗi tập hợp $M$, kí hiệu $\mathcal{P}(M)$ là họ tất cả tập con của $M$ (như vậy $|\mathcal{P}(M)|=2^{|M|}$).

 

Với mỗi tập con $A$ của $X$, ta phân hoạch $A=C\cup L$, trong đó $C$ gồm các phần tử chẳn và $L$ gồm các phần tử lẻ.

Với tập hợp $C$, ta thiết lập ánh xạ $\mathcal{P}\Big(\{2,4,\dots,2n-2,2n\}\Big)\to \mathcal{P}\Big(\{1,2,\dots,n\}\Big)$ như sau

\[C=\{c_1,c_2,\dots,c_k\}\longmapsto \underbrace{\{2,4,\dots,2n-2,2n\}\setminus\{c_1,c_2,\dots,c_k\}}_{\{d_1,d_2,\dots,d_{n-k}\}}\longmapsto \left \{ \frac{d_1}{2},\frac{d_2}{2},\dots,\frac{d_{n-k}}{2} \right \}.\]

Còn đối với tập $L$, ta thiết lập ánh xạ $\mathcal{P}\Big(\{1,3,\dots,2n-3,2n-1\}\Big)\to \mathcal{P}\Big(\{n+1,n+2,\dots,2n\}\Big)$ như sau

\[L=\{l_1,l_2,\dots,l_k\}\longmapsto \left \{ \frac{l_1+1}{2}+n,\frac{l_2+1}{2}+n,\dots,\frac{l_{k}+1}{2}+n \right \}.\]

Như vậy ta đã xây dựng một ánh xạ $X\to Y$ với

\[A=\{c_1,c_2,\dots,c_k,l_1,l_2,\dots,l_k\}\longmapsto \left \{ \frac{d_1}{2},\frac{d_2}{2},\dots,\frac{d_{n-k}}{2}, \frac{l_1+1}{2}+n,\frac{l_2+1}{2}+n,\dots,\frac{l_{k}+1}{2}+n \right \}.\]

Để chứng minh đây là một song ánh thì chỉ cần chứng tỏ mỗi bước thiết lập là song ánh, hoặc chứng tỏ $|X|=|Y|$ tương đương với

\[\sum_{k=0}^n\binom{n}{k}^2=\binom{2n}{n}.\]

 

 

Ghi chú. Một số bài xây dựng song ánh có thể kể đến GGTH 2017, Ấn Độ 2013 hoặc ở đây.